LSAT and Law School Admissions Forum

Get expert LSAT preparation and law school admissions advice from PowerScore Test Preparation.

 srcline@noctrl.edu
  • Posts: 243
  • Joined: Oct 16, 2015
|
#33266
Hello

So with these types of questions you're trying to find the a.c. that makes the conclusion 100 % true. So the conclusion is

"therefore, the witness's testimony should be excluded". I picked E for this because in the stimulus "yet the witness claims to recognize the assailant, but no my famous client...... Doesnt E establish that the witness could be lying.....?

If someone can explain D and E that would be helpful.

Thankyou
Sarah
 Emily Haney-Caron
PowerScore Staff
  • PowerScore Staff
  • Posts: 577
  • Joined: Jan 12, 2012
|
#33333
Hi Sarah,

Take a look again. The conclusion doesn't require that the witness is lying; only that the testimony should be excluded. D provides conditional reasoning, that looks like this:
witness claims to recognize both parties involved in the assault :arrow: testimony included
witness claims to recognize both parties involved in the assault :arrow: testimony included

See how that means that the testimony should be excluded?
 jgabalski
  • Posts: 16
  • Joined: Feb 16, 2017
|
#33485
Can someone please help me differentiate between answer choices A and D here? When doing the question originally, I failed to identify this as a justify the conclusion question. I incorrectly choose choice E, which seems like more of a natural assumption, rather than something that proves the conclusion 100%. However, when redoing the question, I was still confused between the logical difference between A and D. Thank you.
 Charlie Melman
PowerScore Staff
  • PowerScore Staff
  • Posts: 85
  • Joined: Feb 10, 2017
|
#33504
jgabalski wrote:Can someone please help me differentiate between answer choices A and D here? When doing the question originally, I failed to identify this as a justify the conclusion question. I incorrectly choose choice E, which seems like more of a natural assumption, rather than something that proves the conclusion 100%. However, when redoing the question, I was still confused between the logical difference between A and D. Thank you.
Hi J,

The stimulus essentially says that the witness's testimony should not be included because the witness only recognized one of the two parties. That can be diagrammed as:

(If doesn't recognize both) :arrow: (Don't include testimony)

Answer choice (A) says:

(If does recognize both) :arrow: (Do include testimony)

Do you see why this is a Mistaken Negation? Negating the sufficient condition says nothing about the necessary condition.

Answer choice (D) says:

(If include testimony) :arrow: (Does recognize both)

Notice how this is the contrapositive of the argument in the stimulus. Thus, it says exactly the same thing, but in a different order. Because we're entitled to assume that each answer choice is true, this simply validates the premise on which the argument rests, and which the argument never justified. So this answer choice essentially says, "Yes, the premise in the argument is justified."

I hope this helps!
 Leela
  • Posts: 63
  • Joined: Apr 13, 2019
|
#64697
Could someone please explain the prephrase for this stimulus? Also, I understand why D justifies the conclusion, but I'm not understanding why D is a better answer choice than E.
 Adam Tyson
PowerScore Staff
  • PowerScore Staff
  • Posts: 5153
  • Joined: Apr 14, 2011
|
#64733
The problem with answer E, Leela, is that it does nothing to support the conclusion. So what if it is unlikely that someone would fail to recognize the famous client? How does that prove that the testimony should be excluded? That is the claim we need to justify here, so the correct answer has to deal with either excluding or including that testimony. Answer E never addresses the issue of whether the testimony should be excluded or included, so it's a loser.
User avatar
 PresidentLSAT
  • Posts: 87
  • Joined: Apr 19, 2021
|
#99500
For this question, I predicted,

We should exclude the testimony of a witness if they claimed to acknowledge only the assailant.

E was a bit iffy for me but I went with it anyway.

I ruled out D because (maybe I'm not explaining this correctly) I tend not to argue the opposite. I know the conditions that do not allow the testimony to be included. Can't fully say I know when it should be included. The patterns get confusing the more familiar I become with this test. Any advice will be appreciated.
User avatar
 Jeff Wren
PowerScore Staff
  • PowerScore Staff
  • Posts: 389
  • Joined: Oct 19, 2022
|
#99658
Hi PresidentLSAT,

So the first thing to notice is that this is a Justify question, which can be thought of as a 100% strengthen question. Justify questions often (but not always) involve conditional reasoning.

The basic argument goes:

Premise: The witness didn't recognize my famous client.

Conclusion: The witness's testimony should be excluded.

A straightforward prephrase that 100% gets us from the premise to the conclusion would be:

If a witness doesn't recognize all of the parties involved in an incident, then that witness's testimony should be excluded.

If we added that statement into the argument, it would 100% prove the conclusion (since the witness here doesn't recognize all of the parties involved, that gets us to the conclusion that their testimony should be excluded).

That conditional statement can be stated in two ways; however, the "original" as it is written above or its contrapositive. If you're not familiar with contrapositives, then you definitely will want to read up on them as they are often tested on the LSAT. (They are covered in Lesson 2 of our LSAT course and Chapter 6 of The Logical Reasoning Bible.) Contrapositives are not opposites; they are actually identical in meaning to the original conditional statement. They are like two different sides of the same coin. They look different but mean the same thing.

Here, the contrapositive of our original prephrase would be:

If a witness's testimony should NOT be excluded (which is just another way of saying it should be included), then that witness does recognize all of the parties involved in an incident.

Answer D is a basically this contrapositive just using slightly different wording.

Get the most out of your LSAT Prep Plus subscription.

Analyze and track your performance with our Testing and Analytics Package.